Mathematical induction - recursionProve the inequality $n! geq 2^n$ by inductionReason behind particular...

What is the offset in a seaplane's hull?

What do you call something that goes against the spirit of the law, but is legal when interpreting the law to the letter?

How to type dʒ symbol (IPA) on Mac?

What defenses are there against being summoned by the Gate spell?

A function which translates a sentence to title-case

If Manufacturer spice model and Datasheet give different values which should I use?

DOS, create pipe for stdin/stdout of command.com(or 4dos.com) in C or Batch?

What would the Romans have called "sorcery"?

Can Medicine checks be used, with decent rolls, to completely mitigate the risk of death from ongoing damage?

Can I interfere when another PC is about to be attacked?

What do you call a Matrix-like slowdown and camera movement effect?

What are these boxed doors outside store fronts in New York?

How to calculate implied correlation via observed market price (Margrabe option)

Can an x86 CPU running in real mode be considered to be basically an 8086 CPU?

Can a German sentence have two subjects?

Why doesn't Newton's third law mean a person bounces back to where they started when they hit the ground?

How to make payment on the internet without leaving a money trail?

whey we use polarized capacitor?

Example of a relative pronoun

How can I fix this gap between bookcases I made?

Chess with symmetric move-square

What Brexit solution does the DUP want?

Why don't electron-positron collisions release infinite energy?

Circuitry of TV splitters



Mathematical induction - recursion


Prove the inequality $n! geq 2^n$ by inductionReason behind particular induction stepMathematical induction for inequalities with a constant at the right sideApplications of mathematical inductionMathematical induction--When it can and can't be usedHow have they done the algebra here?In-Depth Explanation of How to Do Mathematical Induction Over the Set $mathbb{R}$ of All Real Numbers?Interpretation of 2 proofs involving limits at infinity and mathematical inductionShow that an inequality is true using mathematical induction and the mean value theoremOn Assumptions In Induction













0












$begingroup$


I am struggling with that exercise... i don't understand a step in the answer. Maybe somebody Can explain in here or at least give me a hint what's going on there.



$a_{n} = frac{7*(n+2)}{2*n} *a_{n-1}$ ,$a_{1} = 14$



So it's...



$a_{2} = frac{7*4}{4} * a_{1} = frac{7*4 * 14}{4} = frac{2* 7^{2}*4 }{4}$



$a_{3} = frac{7*5}{6} * a_{2} = frac{2* 7^{3}*4*5 }{4*6}$



And here is the problem...
a4 equals:



$a_{4} = frac{7*6}{8} * a_{3} = frac{2* 7^{4}*4*5*6 }{4*6*8}$



We are multiplying numerator and denominator with 2*3 to get factorial 6! on numerator. I can't understand how the
$(1*2*3)^2$ got on denominator and how after that we transformed it to $3! * 4!$. Next steps are quite easy and the mathematical induction is fully understandable for me.



$a_{4} = frac{2* 7^{4}*6! }{ (1*2*3)^{2} * 2 *3 *4 } = frac{2* 7^{4}*6! }{ 3! 4! }$



Full answer of the exercise: JPG image










share|cite|improve this question









$endgroup$

















    0












    $begingroup$


    I am struggling with that exercise... i don't understand a step in the answer. Maybe somebody Can explain in here or at least give me a hint what's going on there.



    $a_{n} = frac{7*(n+2)}{2*n} *a_{n-1}$ ,$a_{1} = 14$



    So it's...



    $a_{2} = frac{7*4}{4} * a_{1} = frac{7*4 * 14}{4} = frac{2* 7^{2}*4 }{4}$



    $a_{3} = frac{7*5}{6} * a_{2} = frac{2* 7^{3}*4*5 }{4*6}$



    And here is the problem...
    a4 equals:



    $a_{4} = frac{7*6}{8} * a_{3} = frac{2* 7^{4}*4*5*6 }{4*6*8}$



    We are multiplying numerator and denominator with 2*3 to get factorial 6! on numerator. I can't understand how the
    $(1*2*3)^2$ got on denominator and how after that we transformed it to $3! * 4!$. Next steps are quite easy and the mathematical induction is fully understandable for me.



    $a_{4} = frac{2* 7^{4}*6! }{ (1*2*3)^{2} * 2 *3 *4 } = frac{2* 7^{4}*6! }{ 3! 4! }$



    Full answer of the exercise: JPG image










    share|cite|improve this question









    $endgroup$















      0












      0








      0





      $begingroup$


      I am struggling with that exercise... i don't understand a step in the answer. Maybe somebody Can explain in here or at least give me a hint what's going on there.



      $a_{n} = frac{7*(n+2)}{2*n} *a_{n-1}$ ,$a_{1} = 14$



      So it's...



      $a_{2} = frac{7*4}{4} * a_{1} = frac{7*4 * 14}{4} = frac{2* 7^{2}*4 }{4}$



      $a_{3} = frac{7*5}{6} * a_{2} = frac{2* 7^{3}*4*5 }{4*6}$



      And here is the problem...
      a4 equals:



      $a_{4} = frac{7*6}{8} * a_{3} = frac{2* 7^{4}*4*5*6 }{4*6*8}$



      We are multiplying numerator and denominator with 2*3 to get factorial 6! on numerator. I can't understand how the
      $(1*2*3)^2$ got on denominator and how after that we transformed it to $3! * 4!$. Next steps are quite easy and the mathematical induction is fully understandable for me.



      $a_{4} = frac{2* 7^{4}*6! }{ (1*2*3)^{2} * 2 *3 *4 } = frac{2* 7^{4}*6! }{ 3! 4! }$



      Full answer of the exercise: JPG image










      share|cite|improve this question









      $endgroup$




      I am struggling with that exercise... i don't understand a step in the answer. Maybe somebody Can explain in here or at least give me a hint what's going on there.



      $a_{n} = frac{7*(n+2)}{2*n} *a_{n-1}$ ,$a_{1} = 14$



      So it's...



      $a_{2} = frac{7*4}{4} * a_{1} = frac{7*4 * 14}{4} = frac{2* 7^{2}*4 }{4}$



      $a_{3} = frac{7*5}{6} * a_{2} = frac{2* 7^{3}*4*5 }{4*6}$



      And here is the problem...
      a4 equals:



      $a_{4} = frac{7*6}{8} * a_{3} = frac{2* 7^{4}*4*5*6 }{4*6*8}$



      We are multiplying numerator and denominator with 2*3 to get factorial 6! on numerator. I can't understand how the
      $(1*2*3)^2$ got on denominator and how after that we transformed it to $3! * 4!$. Next steps are quite easy and the mathematical induction is fully understandable for me.



      $a_{4} = frac{2* 7^{4}*6! }{ (1*2*3)^{2} * 2 *3 *4 } = frac{2* 7^{4}*6! }{ 3! 4! }$



      Full answer of the exercise: JPG image







      induction recursion






      share|cite|improve this question













      share|cite|improve this question











      share|cite|improve this question




      share|cite|improve this question










      asked Mar 20 at 12:14









      Jim.DJim.D

      31




      31






















          3 Answers
          3






          active

          oldest

          votes


















          0












          $begingroup$

          Your doubt is valid:
          $$begin{align}a_{n}& = frac{7*(n+2)}{2*n} *a_{n-1}, a_1=14\
          a_2&=frac{7*4}{4}*a_1=frac{2*7^2*4}{4}=frac{2*7^2*4!}{4!}\
          a_3&=frac{7*5}{6}*a_2=frac{2*7^3*4*5}{4*6}=frac{7^3*5!}{2*3!*3!}\
          a_4&=frac{7*6}{8}*a_3=frac{7^4*5!*6}{2*3!*3!*8}=frac{7^4*6!}{2^2*3!*4!}\
          a_5&=frac{7*7}{10}*a_4=frac{7^5*6!*7}{2^2*3!*4!*10}=frac{7^5*7!}{2^3*3!*5!}\
          vdots\
          a_n&=frac{7^n*(n+2)!}{2^{n-2}*3!*n!},nge 1.end{align}$$

          Verify:
          $$begin{align}a_1&=frac{7^1*(1+2)!}{2^{1-2}*3!*1!}=14\
          a_2&=frac{7^2*(2+2)!}{2^{2-2}*3!*2!}=2*7^2\
          a_3&=frac{7^3*(3+2)!}{2^{3-2}*3!*3!}=frac{7^3*5!}{2*3!*3!}end{align}$$






          share|cite|improve this answer









          $endgroup$













          • $begingroup$
            Thank u a lot. Now is everything clear! Thanks once again :)
            $endgroup$
            – Jim.D
            Mar 21 at 14:10










          • $begingroup$
            You are welcome. If you think it answers your question, you can accept it. Good luck
            $endgroup$
            – farruhota
            Mar 21 at 14:24



















          0












          $begingroup$

          I got $$a_n=frac{2}{3}frac{7^n}{2^n}(n+1)(n+2)$$






          share|cite|improve this answer









          $endgroup$





















            0












            $begingroup$

            $$a_n=a_{n-r}cdotdfrac{7^r(n+2)(n+1)cdots(n-r+3)}{2^r n(n-1)cdots(n-r+1)}$$ $0le rle n-1$



            For $rge2implies nge3$ $$a_n=a_{n-r}cdotleft(dfrac72right)^rdfrac{(n+2)(n+1)}{(n-r+2)(n-r+1)}$$



            Set $r=n-1$






            share|cite|improve this answer











            $endgroup$














              Your Answer





              StackExchange.ifUsing("editor", function () {
              return StackExchange.using("mathjaxEditing", function () {
              StackExchange.MarkdownEditor.creationCallbacks.add(function (editor, postfix) {
              StackExchange.mathjaxEditing.prepareWmdForMathJax(editor, postfix, [["$", "$"], ["\\(","\\)"]]);
              });
              });
              }, "mathjax-editing");

              StackExchange.ready(function() {
              var channelOptions = {
              tags: "".split(" "),
              id: "69"
              };
              initTagRenderer("".split(" "), "".split(" "), channelOptions);

              StackExchange.using("externalEditor", function() {
              // Have to fire editor after snippets, if snippets enabled
              if (StackExchange.settings.snippets.snippetsEnabled) {
              StackExchange.using("snippets", function() {
              createEditor();
              });
              }
              else {
              createEditor();
              }
              });

              function createEditor() {
              StackExchange.prepareEditor({
              heartbeatType: 'answer',
              autoActivateHeartbeat: false,
              convertImagesToLinks: true,
              noModals: true,
              showLowRepImageUploadWarning: true,
              reputationToPostImages: 10,
              bindNavPrevention: true,
              postfix: "",
              imageUploader: {
              brandingHtml: "Powered by u003ca class="icon-imgur-white" href="https://imgur.com/"u003eu003c/au003e",
              contentPolicyHtml: "User contributions licensed under u003ca href="https://creativecommons.org/licenses/by-sa/3.0/"u003ecc by-sa 3.0 with attribution requiredu003c/au003e u003ca href="https://stackoverflow.com/legal/content-policy"u003e(content policy)u003c/au003e",
              allowUrls: true
              },
              noCode: true, onDemand: true,
              discardSelector: ".discard-answer"
              ,immediatelyShowMarkdownHelp:true
              });


              }
              });














              draft saved

              draft discarded


















              StackExchange.ready(
              function () {
              StackExchange.openid.initPostLogin('.new-post-login', 'https%3a%2f%2fmath.stackexchange.com%2fquestions%2f3155370%2fmathematical-induction-recursion%23new-answer', 'question_page');
              }
              );

              Post as a guest















              Required, but never shown

























              3 Answers
              3






              active

              oldest

              votes








              3 Answers
              3






              active

              oldest

              votes









              active

              oldest

              votes






              active

              oldest

              votes









              0












              $begingroup$

              Your doubt is valid:
              $$begin{align}a_{n}& = frac{7*(n+2)}{2*n} *a_{n-1}, a_1=14\
              a_2&=frac{7*4}{4}*a_1=frac{2*7^2*4}{4}=frac{2*7^2*4!}{4!}\
              a_3&=frac{7*5}{6}*a_2=frac{2*7^3*4*5}{4*6}=frac{7^3*5!}{2*3!*3!}\
              a_4&=frac{7*6}{8}*a_3=frac{7^4*5!*6}{2*3!*3!*8}=frac{7^4*6!}{2^2*3!*4!}\
              a_5&=frac{7*7}{10}*a_4=frac{7^5*6!*7}{2^2*3!*4!*10}=frac{7^5*7!}{2^3*3!*5!}\
              vdots\
              a_n&=frac{7^n*(n+2)!}{2^{n-2}*3!*n!},nge 1.end{align}$$

              Verify:
              $$begin{align}a_1&=frac{7^1*(1+2)!}{2^{1-2}*3!*1!}=14\
              a_2&=frac{7^2*(2+2)!}{2^{2-2}*3!*2!}=2*7^2\
              a_3&=frac{7^3*(3+2)!}{2^{3-2}*3!*3!}=frac{7^3*5!}{2*3!*3!}end{align}$$






              share|cite|improve this answer









              $endgroup$













              • $begingroup$
                Thank u a lot. Now is everything clear! Thanks once again :)
                $endgroup$
                – Jim.D
                Mar 21 at 14:10










              • $begingroup$
                You are welcome. If you think it answers your question, you can accept it. Good luck
                $endgroup$
                – farruhota
                Mar 21 at 14:24
















              0












              $begingroup$

              Your doubt is valid:
              $$begin{align}a_{n}& = frac{7*(n+2)}{2*n} *a_{n-1}, a_1=14\
              a_2&=frac{7*4}{4}*a_1=frac{2*7^2*4}{4}=frac{2*7^2*4!}{4!}\
              a_3&=frac{7*5}{6}*a_2=frac{2*7^3*4*5}{4*6}=frac{7^3*5!}{2*3!*3!}\
              a_4&=frac{7*6}{8}*a_3=frac{7^4*5!*6}{2*3!*3!*8}=frac{7^4*6!}{2^2*3!*4!}\
              a_5&=frac{7*7}{10}*a_4=frac{7^5*6!*7}{2^2*3!*4!*10}=frac{7^5*7!}{2^3*3!*5!}\
              vdots\
              a_n&=frac{7^n*(n+2)!}{2^{n-2}*3!*n!},nge 1.end{align}$$

              Verify:
              $$begin{align}a_1&=frac{7^1*(1+2)!}{2^{1-2}*3!*1!}=14\
              a_2&=frac{7^2*(2+2)!}{2^{2-2}*3!*2!}=2*7^2\
              a_3&=frac{7^3*(3+2)!}{2^{3-2}*3!*3!}=frac{7^3*5!}{2*3!*3!}end{align}$$






              share|cite|improve this answer









              $endgroup$













              • $begingroup$
                Thank u a lot. Now is everything clear! Thanks once again :)
                $endgroup$
                – Jim.D
                Mar 21 at 14:10










              • $begingroup$
                You are welcome. If you think it answers your question, you can accept it. Good luck
                $endgroup$
                – farruhota
                Mar 21 at 14:24














              0












              0








              0





              $begingroup$

              Your doubt is valid:
              $$begin{align}a_{n}& = frac{7*(n+2)}{2*n} *a_{n-1}, a_1=14\
              a_2&=frac{7*4}{4}*a_1=frac{2*7^2*4}{4}=frac{2*7^2*4!}{4!}\
              a_3&=frac{7*5}{6}*a_2=frac{2*7^3*4*5}{4*6}=frac{7^3*5!}{2*3!*3!}\
              a_4&=frac{7*6}{8}*a_3=frac{7^4*5!*6}{2*3!*3!*8}=frac{7^4*6!}{2^2*3!*4!}\
              a_5&=frac{7*7}{10}*a_4=frac{7^5*6!*7}{2^2*3!*4!*10}=frac{7^5*7!}{2^3*3!*5!}\
              vdots\
              a_n&=frac{7^n*(n+2)!}{2^{n-2}*3!*n!},nge 1.end{align}$$

              Verify:
              $$begin{align}a_1&=frac{7^1*(1+2)!}{2^{1-2}*3!*1!}=14\
              a_2&=frac{7^2*(2+2)!}{2^{2-2}*3!*2!}=2*7^2\
              a_3&=frac{7^3*(3+2)!}{2^{3-2}*3!*3!}=frac{7^3*5!}{2*3!*3!}end{align}$$






              share|cite|improve this answer









              $endgroup$



              Your doubt is valid:
              $$begin{align}a_{n}& = frac{7*(n+2)}{2*n} *a_{n-1}, a_1=14\
              a_2&=frac{7*4}{4}*a_1=frac{2*7^2*4}{4}=frac{2*7^2*4!}{4!}\
              a_3&=frac{7*5}{6}*a_2=frac{2*7^3*4*5}{4*6}=frac{7^3*5!}{2*3!*3!}\
              a_4&=frac{7*6}{8}*a_3=frac{7^4*5!*6}{2*3!*3!*8}=frac{7^4*6!}{2^2*3!*4!}\
              a_5&=frac{7*7}{10}*a_4=frac{7^5*6!*7}{2^2*3!*4!*10}=frac{7^5*7!}{2^3*3!*5!}\
              vdots\
              a_n&=frac{7^n*(n+2)!}{2^{n-2}*3!*n!},nge 1.end{align}$$

              Verify:
              $$begin{align}a_1&=frac{7^1*(1+2)!}{2^{1-2}*3!*1!}=14\
              a_2&=frac{7^2*(2+2)!}{2^{2-2}*3!*2!}=2*7^2\
              a_3&=frac{7^3*(3+2)!}{2^{3-2}*3!*3!}=frac{7^3*5!}{2*3!*3!}end{align}$$







              share|cite|improve this answer












              share|cite|improve this answer



              share|cite|improve this answer










              answered Mar 20 at 13:20









              farruhotafarruhota

              21.8k2842




              21.8k2842












              • $begingroup$
                Thank u a lot. Now is everything clear! Thanks once again :)
                $endgroup$
                – Jim.D
                Mar 21 at 14:10










              • $begingroup$
                You are welcome. If you think it answers your question, you can accept it. Good luck
                $endgroup$
                – farruhota
                Mar 21 at 14:24


















              • $begingroup$
                Thank u a lot. Now is everything clear! Thanks once again :)
                $endgroup$
                – Jim.D
                Mar 21 at 14:10










              • $begingroup$
                You are welcome. If you think it answers your question, you can accept it. Good luck
                $endgroup$
                – farruhota
                Mar 21 at 14:24
















              $begingroup$
              Thank u a lot. Now is everything clear! Thanks once again :)
              $endgroup$
              – Jim.D
              Mar 21 at 14:10




              $begingroup$
              Thank u a lot. Now is everything clear! Thanks once again :)
              $endgroup$
              – Jim.D
              Mar 21 at 14:10












              $begingroup$
              You are welcome. If you think it answers your question, you can accept it. Good luck
              $endgroup$
              – farruhota
              Mar 21 at 14:24




              $begingroup$
              You are welcome. If you think it answers your question, you can accept it. Good luck
              $endgroup$
              – farruhota
              Mar 21 at 14:24











              0












              $begingroup$

              I got $$a_n=frac{2}{3}frac{7^n}{2^n}(n+1)(n+2)$$






              share|cite|improve this answer









              $endgroup$


















                0












                $begingroup$

                I got $$a_n=frac{2}{3}frac{7^n}{2^n}(n+1)(n+2)$$






                share|cite|improve this answer









                $endgroup$
















                  0












                  0








                  0





                  $begingroup$

                  I got $$a_n=frac{2}{3}frac{7^n}{2^n}(n+1)(n+2)$$






                  share|cite|improve this answer









                  $endgroup$



                  I got $$a_n=frac{2}{3}frac{7^n}{2^n}(n+1)(n+2)$$







                  share|cite|improve this answer












                  share|cite|improve this answer



                  share|cite|improve this answer










                  answered Mar 20 at 12:33









                  Dr. Sonnhard GraubnerDr. Sonnhard Graubner

                  78.7k42867




                  78.7k42867























                      0












                      $begingroup$

                      $$a_n=a_{n-r}cdotdfrac{7^r(n+2)(n+1)cdots(n-r+3)}{2^r n(n-1)cdots(n-r+1)}$$ $0le rle n-1$



                      For $rge2implies nge3$ $$a_n=a_{n-r}cdotleft(dfrac72right)^rdfrac{(n+2)(n+1)}{(n-r+2)(n-r+1)}$$



                      Set $r=n-1$






                      share|cite|improve this answer











                      $endgroup$


















                        0












                        $begingroup$

                        $$a_n=a_{n-r}cdotdfrac{7^r(n+2)(n+1)cdots(n-r+3)}{2^r n(n-1)cdots(n-r+1)}$$ $0le rle n-1$



                        For $rge2implies nge3$ $$a_n=a_{n-r}cdotleft(dfrac72right)^rdfrac{(n+2)(n+1)}{(n-r+2)(n-r+1)}$$



                        Set $r=n-1$






                        share|cite|improve this answer











                        $endgroup$
















                          0












                          0








                          0





                          $begingroup$

                          $$a_n=a_{n-r}cdotdfrac{7^r(n+2)(n+1)cdots(n-r+3)}{2^r n(n-1)cdots(n-r+1)}$$ $0le rle n-1$



                          For $rge2implies nge3$ $$a_n=a_{n-r}cdotleft(dfrac72right)^rdfrac{(n+2)(n+1)}{(n-r+2)(n-r+1)}$$



                          Set $r=n-1$






                          share|cite|improve this answer











                          $endgroup$



                          $$a_n=a_{n-r}cdotdfrac{7^r(n+2)(n+1)cdots(n-r+3)}{2^r n(n-1)cdots(n-r+1)}$$ $0le rle n-1$



                          For $rge2implies nge3$ $$a_n=a_{n-r}cdotleft(dfrac72right)^rdfrac{(n+2)(n+1)}{(n-r+2)(n-r+1)}$$



                          Set $r=n-1$







                          share|cite|improve this answer














                          share|cite|improve this answer



                          share|cite|improve this answer








                          edited Mar 20 at 12:41

























                          answered Mar 20 at 12:32









                          lab bhattacharjeelab bhattacharjee

                          228k15158279




                          228k15158279






























                              draft saved

                              draft discarded




















































                              Thanks for contributing an answer to Mathematics Stack Exchange!


                              • Please be sure to answer the question. Provide details and share your research!

                              But avoid



                              • Asking for help, clarification, or responding to other answers.

                              • Making statements based on opinion; back them up with references or personal experience.


                              Use MathJax to format equations. MathJax reference.


                              To learn more, see our tips on writing great answers.




                              draft saved


                              draft discarded














                              StackExchange.ready(
                              function () {
                              StackExchange.openid.initPostLogin('.new-post-login', 'https%3a%2f%2fmath.stackexchange.com%2fquestions%2f3155370%2fmathematical-induction-recursion%23new-answer', 'question_page');
                              }
                              );

                              Post as a guest















                              Required, but never shown





















































                              Required, but never shown














                              Required, but never shown












                              Required, but never shown







                              Required, but never shown

































                              Required, but never shown














                              Required, but never shown












                              Required, but never shown







                              Required, but never shown







                              Popular posts from this blog

                              Magento 2 - Add success message with knockout Planned maintenance scheduled April 23, 2019 at 23:30 UTC (7:30pm US/Eastern) Announcing the arrival of Valued Associate #679: Cesar Manara Unicorn Meta Zoo #1: Why another podcast?Success / Error message on ajax request$.widget is not a function when loading a homepage after add custom jQuery on custom themeHow can bind jQuery to current document in Magento 2 When template load by ajaxRedirect page using plugin in Magento 2Magento 2 - Update quantity and totals of cart page without page reload?Magento 2: Quote data not loaded on knockout checkoutMagento 2 : I need to change add to cart success message after adding product into cart through pluginMagento 2.2.5 How to add additional products to cart from new checkout step?Magento 2 Add error/success message with knockoutCan't validate Post Code on checkout page

                              Fil:Tokke komm.svg

                              Where did Arya get these scars? Unicorn Meta Zoo #1: Why another podcast? Announcing the arrival of Valued Associate #679: Cesar Manara Favourite questions and answers from the 1st quarter of 2019Why did Arya refuse to end it?Has the pronunciation of Arya Stark's name changed?Has Arya forgiven people?Why did Arya Stark lose her vision?Why can Arya still use the faces?Has the Narrow Sea become narrower?Does Arya Stark know how to make poisons outside of the House of Black and White?Why did Nymeria leave Arya?Why did Arya not kill the Lannister soldiers she encountered in the Riverlands?What is the current canonical age of Sansa, Bran and Arya Stark?